Bạn chưa đăng nhập. Vui lòng đăng nhập để hỏi bài

Những câu hỏi liên quan
ngọc linh
Xem chi tiết
Nguyễn Hoàng Minh
28 tháng 11 2021 lúc 9:09

Ta có \(x+y\le1\Leftrightarrow1-x\ge y>0\Leftrightarrow0< x< 1\)

Giả sử \(x^2-\dfrac{3}{4x}-\dfrac{x}{y}\le-\dfrac{9}{4}\)

\(\Leftrightarrow4x^2+9\le\dfrac{3}{x}+\dfrac{4x}{y}\\ \Leftrightarrow\dfrac{4x}{1-x}+\dfrac{3}{x}\ge4x^2+9\\ \Leftrightarrow\dfrac{4x^2+3\left(1-x\right)-x\left(4x^2+9\right)\left(1-x\right)}{x\left(1-x\right)}\ge0\\ \Leftrightarrow\dfrac{4x^4-4x^3+13x^2-12x+3}{x\left(1-x\right)}\ge0\\ \Leftrightarrow\dfrac{\left(x^2+3\right)\left(2x-1\right)^2}{x\left(1-x\right)}\ge0\)

Vì \(x>0;1-x>0\) nên BĐT trên luôn đúng

Vậy ta được đpcm

Dấu \("="\Leftrightarrow x=y=\dfrac{1}{2}\)

Lấp La Lấp Lánh
Xem chi tiết
Trần Tuấn Hoàng
Xem chi tiết
Người Vô Danh
28 tháng 2 2022 lúc 22:48

\(\left(x+y+z\right)^2=x^2+y^2+z^2+2xy+2xz+2yz=z^2+\left(x+y\right)^2+2z\left(x+y\right)=36\)

áp dụng BĐT cosi : 

\(z^2+\left(x+y\right)^2\ge2z\left(x+y\right)\)

<=> \(z^2+\left(x+y\right)^2+2z\left(x+y\right)\ge4z\left(x+y\right)=36< =>z\left(x+y\right)\ge9\)

ta lại có \(\dfrac{x+y}{xyz}=\dfrac{x}{xyz}+\dfrac{y}{xyz}=\dfrac{1}{yz}+\dfrac{1}{xz}\) áp dụng BĐT buhihacopxki dạng phân thức => \(\dfrac{1}{yz}+\dfrac{1}{xz}\ge\dfrac{4}{yz+xz}=\dfrac{4}{z\left(x+y\right)}\ge\dfrac{4}{9}\left(đpcm\right)\)

dấu bằng xảy ra khi \(\left[{}\begin{matrix}yz=xz< =>x=y\\x+y+z=6\\z^2=\left(x+y\right)^2\end{matrix}\right.< =>\left[{}\begin{matrix}x+y+z=6\\z=2x=2y\end{matrix}\right.< =>\left[{}\begin{matrix}x=y=\dfrac{3}{2}\\z=3\end{matrix}\right.\)

Hồ Nhật Phi
28 tháng 2 2022 lúc 23:00

\(\dfrac{x+y}{xyz}=\dfrac{1}{yz}+\dfrac{1}{xz}\).

Áp dụng bất đẳng thức Cauchy-Schawrz dạng Engel:

\(\dfrac{1}{yz}+\dfrac{1}{xz}\ge\dfrac{4}{z\left(x+y\right)}\)     (1).

Áp dụng bất đẳng thức Cauchy cho hai số dương z và x+y, ta có:

\(z\left(x+y\right)\le\left(\dfrac{x+y+z}{2}\right)^2=9\). Suy ra, \(\dfrac{4}{z\left(x+y\right)}\ge\dfrac{4}{9}\)     (2).

Từ (1) và (2), suy ra \(\dfrac{x+y}{xyz}\ge\dfrac{4}{9}\) (đpcm).

Dấu "=" xảy ra khi và chỉ khi \(\dfrac{1}{yz}=\dfrac{1}{xz}\) và \(z=x+y\).

 

Nguyễn An
Xem chi tiết
Trần Tiến Đạt
Xem chi tiết
OH-YEAH^^
8 tháng 6 2021 lúc 20:13

a,\(\dfrac{6}{2x+1}=\dfrac{2}{7}\)

\(\dfrac{6}{2x+1}=\dfrac{6}{21}\)

\(2x+1=21\)

\(2x=21-1\)

\(2x=20\)

\(x=10\)

 

đấng ys
Xem chi tiết
Trên con đường thành côn...
24 tháng 8 2021 lúc 11:03

https://olm.vn/hoi-dap/detail/227981379332.html

Bạn tham khảo ở đây nhé.

Hùng Mạnh
Xem chi tiết
Nguyễn Tuấn Minh
Xem chi tiết
 Mashiro Shiina
6 tháng 11 2018 lúc 21:24

\(6\left(x^2+y^2+z^2\right)+10\left(xy+yz+xz\right)+2\left(\dfrac{1}{2x+y+z}+\dfrac{1}{x+2y+z}+\dfrac{1}{x+y+2z}\right)\)

\(=6\left(x^2+y^2+z^2\right)+12\left(xy+yz+xz\right)+2\left(\dfrac{1}{2x+y+z}+\dfrac{1}{x+2y+z}+\dfrac{1}{x+y+2z}\right)-2\left(xy+yz+xz\right)\)

\(=6\left(x+y+z\right)^2+2\left(\dfrac{1}{2x+y+z}+\dfrac{1}{x+2y+z}+\dfrac{1}{2z+x+y}\right)-2\left(xy+yz+xz\right)\)

\(\ge6\left(x+y+z\right)^2+2.\dfrac{\left(1+1+1\right)^2}{2x+y+z+x+2y+z+2z+x+y}-2\left(xy+yz+xz\right)\)

\(=6\left(x+y+z\right)^2+\dfrac{18}{4\left(x+y+z\right)}-2\left(xy+yz+xz\right)\)

\(\ge6\left(x+y+z\right)^2+\dfrac{18}{4\left(x+y+z\right)}-\dfrac{2}{3}\left(x+y+z\right)^2\)

\(=6.\left(\dfrac{3}{4}\right)^2+\dfrac{18}{4.\dfrac{3}{4}}-\dfrac{2}{3}.\left(\dfrac{3}{4}\right)^2=9\)

\("="\Leftrightarrow x=y=z=\dfrac{1}{4}\)

Diệp Kì Thiên
6 tháng 11 2018 lúc 21:18

a) ab+bc+ca\(\le\dfrac{\left(a+c+b\right)^2}{3}\)

\(\Leftrightarrow3ab+3bc+3ac\le a^2+b^2+c^2+2ab+2bc+2ac\)

\(\Leftrightarrow ab+bc+ac\le a^2+b^2+c^2\)

\(\Leftrightarrow2ab+2bc+2ca\le2a^2+2b^2+2c^2\)

\(\Leftrightarrow a^2-2ab+b^2+b^2-2bc+c^2+c^2-2ca+a^2\ge0\)

\(\left(a-b\right)^2+\left(b-c\right)^2+\left(c-a\right)^2\ge0\) (luôn đúng \(\forall a,b,c\)

TNA Atula
6 tháng 11 2018 lúc 21:28

a) 3.(ab+bc+ac)≤a2+b2+c2+2ab+2bc+2ac

<=> \(a^2+b^2+c^2-ab-bc-ac\ge0\)

<=> \(2a^2+2b^2+2c^2-2ab-2bc-2ac\ge0\)

<=> (a-b)2+(b-c)2+(a-c)2≥0 ( luon dung voi moi a,b,c)

b) ap dung ket qua tren va vế sau bn xem bài giải của mk ở trên

Vo Thi Minh Dao
Xem chi tiết
 Mashiro Shiina
5 tháng 12 2018 lúc 13:05

Sửa đề nhé\(\dfrac{1}{3x+3y+2z}=\dfrac{1}{\left(z+x\right)+\left(z+y\right)+\left(x+y\right)+\left(x+y\right)}\)

\(\le\dfrac{1}{16}\left(\dfrac{1}{x+z}+\dfrac{1}{z+y}+\dfrac{1}{x+y}+\dfrac{1}{x+y}\right)\)

CMTT và cộng theo vế:

\(VT\le\dfrac{1}{16}\left(\dfrac{1}{x+z}+\dfrac{1}{z+y}+\dfrac{1}{x+y}+\dfrac{1}{x+y}+\dfrac{1}{x+y}+\dfrac{1}{y+z}+\dfrac{1}{x+z}+\dfrac{1}{x+z}+\dfrac{1}{x+z}+\dfrac{1}{x+y}+\dfrac{1}{y+z}+\dfrac{1}{y+z}\right)\)

\(=\dfrac{1}{16}.24=\dfrac{3}{2}\)

\("="\Leftrightarrow x=y=z=\dfrac{1}{4}\)